subject
Business, 07.07.2020 08:01 turner639237

Because chips and salsa are complements, an increase in the price of chips will cause the demand for salsa to decrease. This initial shift in demand for chips results in a higher price for chips; this higher price will cause the demand curve for chips to shift to the right. Which of the following correctly comments on this statement? A) The statement will be true if consumer tastes for chips and salsa do not change.
B) The statement is false because a change in the price of chips would not change the demand for chips.
C) The statement is false because salsa is an inferior good; chips are normal goods.
D) The statement is false because one cannot assume that chips and salsa are complements for all consumers.

ansver
Answers: 1

Another question on Business

question
Business, 22.06.2019 04:00
Assume that the following conditions exist: a. all banks are fully loaned up- there are no excess reserves, and desired excess reserves are always zero. b. the money multiplier is 5 .     c. the planned investment schedule is such that at a 4 percent rate of interest, investment =$1450 billion. at 5 percent, investment is $1420 billion. d. the investment multiplier is 3 . e.. the initial equilibrium level of real gdp is $12 trillion. f. the equilibrium rate of interest is 4 percent now the fed engages in contractionary monetary policy. it sells $1 billion worth of bonds, which reduces the money supply, which in turn raises the market rate of interest by 1 percentage point. calculate the decrease in money supply after fed's sale of bonds: $nothing billion.
Answers: 2
question
Business, 22.06.2019 08:10
What are the period and vertical shift of the cosecant function below? period: ; vertical shift: 1 unit up period: ; vertical shift: 2 units up period: ; vertical shift: 1 unit up period: ; vertical shift: 2 units up?
Answers: 3
question
Business, 22.06.2019 12:00
Areal estate agent is considering changing her cell phone plan. there are three plans to choose from, all of which involve a monthly service charge of $20. plan a has a cost of $.42 a minute for daytime calls and $.17 a minute for evening calls. plan b has a charge of $.52 a minute for daytime calls and $.15 a minute for evening calls. plan c has a flat rate of $80 with 275 minutes of calls allowed per month and a charge of $.38 per minute beyond that, day or evening.a. determine the total charge under each plan for this case: 150 minutes of day calls and 70 minutes of evening calls in a month. (do not round intermediate calculations. round your answer to 2 decimal places. omit the "$" sign in your response.)c. if the agent will use the service for daytime calls, over what range of call minutes will each plan be optimal? (round each answer to the nearest whole number.include the indifference point itself in each answer.)d. suppose that the agent expects both daytime and evening calls. at what point (i.e., percentage of total call minutes used for daytime calls) would she be indifferent between plans a and b?
Answers: 1
question
Business, 23.06.2019 11:00
Jessica thinks that everyone would be better off if financial institutions stopped issuing credit. which statement accurately supports her argument? people would pay less in interest fees. people would have greater protection in case of emergencies. people would need to save for many years to buy a home or open a business. people would support the economy through purchases of more goods and services.
Answers: 1
You know the right answer?
Because chips and salsa are complements, an increase in the price of chips will cause the demand for...
Questions
question
Mathematics, 27.02.2021 02:50
question
Mathematics, 27.02.2021 02:50
question
Health, 27.02.2021 02:50
question
Mathematics, 27.02.2021 02:50
question
Mathematics, 27.02.2021 02:50